Disuguaglianza con [x] e {x}

Polinomi, disuguaglianze, numeri complessi, ...
Rispondi
Avatar utente
jordan
Messaggi: 3988
Iscritto il: 02 feb 2007, 21:19
Località: Pescara
Contatta:

Disuguaglianza con [x] e {x}

Messaggio da jordan »

Sia x un reale maggiore di 1, che non sia intero. Mostrare che
\[ \biggl(\frac{x+\{x\}}{\lfloor x\rfloor}-\frac{\lfloor x\rfloor}{x+\{x\}}\biggr)+\biggl(\frac{x+\lfloor x\rfloor}{\{x\}}-\frac{\{ x\}}{x+\lfloor x\rfloor}\biggr) >\frac{14}{3} \]

(Mediterranean MO 2007)
The only goal of science is the honor of the human spirit.
Avatar utente
petroliopg
Messaggi: 96
Iscritto il: 17 giu 2012, 17:31

Re: Disuguaglianza con [x] e {x}

Messaggio da petroliopg »

non voglio fare figuracce, ma cosa intendi con {x}?
$\displaystyle {x}:= min{n \in \mathbb{Z} : n \ge x}$? non l'avevo mai vista come notazione...
Sensi non ho; né senso. Non ho limite.
Montale

$ \displaystyle i \hbar \dot {\psi} = \hat{H} \psi $
EvaristeG
Site Admin
Messaggi: 4896
Iscritto il: 01 gen 1970, 01:00
Località: Roma
Contatta:

Re: Disuguaglianza con [x] e {x}

Messaggio da EvaristeG »

{x} è la parte frazionaria di x, ovvero x-[x].
Avatar utente
jordan
Messaggi: 3988
Iscritto il: 02 feb 2007, 21:19
Località: Pescara
Contatta:

Re: Disuguaglianza con [x] e {x}

Messaggio da jordan »

petroliopg ha scritto:non voglio fare figuracce, ma cosa intendi con {x}?
$\displaystyle {x}:= min{n \in \mathbb{Z} : n \ge x}$? non l'avevo mai vista come notazione...
Quella che dici si indica (in genere) con $\left\lceil x \right\rceil$

Ps. Quando vuoi mettere le graffe non ti dimenticare gli slash
Text ha scritto:$\text{\{}$
The only goal of science is the honor of the human spirit.
Avatar utente
petroliopg
Messaggi: 96
Iscritto il: 17 giu 2012, 17:31

Re: Disuguaglianza con [x] e {x}

Messaggio da petroliopg »

grazie raga. ci provo ora poi scrivo (o edito questo messaggio)
Sensi non ho; né senso. Non ho limite.
Montale

$ \displaystyle i \hbar \dot {\psi} = \hat{H} \psi $
machete
Messaggi: 52
Iscritto il: 28 ago 2012, 15:44

Re: Disuguaglianza con [x] e {x}

Messaggio da machete »

Ci provo, ma avverto che ci potrebbe essere qualche erroraccio. . . siate clementi! :D
Ricordando che $ x=\left\lfloor x\right\rfloor+\left\{x\right\} $ riscrivo tutti i numeratori e per fare la divisione aggiungo e tolgo ciò che serve, così diventa:

$ \displaystyle \biggl(\frac{\left\lfloor x\right\rfloor+2\{x\}}{\lfloor x\rfloor}+\frac{\{x\}+2\lfloor x\rfloor}{\{x\}}\biggr)-\biggl(\frac{\lfloor x\rfloor+2\{x\}-2\{x\}}{\left\lfloor x\right\rfloor+2\{x\}}+\frac{\{ x\}+2\left\lfloor x\right\rfloor-2\left\lfloor x\right\rfloor}{\{x\}+2\lfloor x\rfloor}\biggr) >\frac{14}{3} $
ossia:
$ \displaystyle 2+2\biggl(\frac{\{x\}}{\lfloor x\rfloor}+\frac{\lfloor x\rfloor}{\{x\}}\biggr)-2+2\biggl(\frac{\left\lfloor x\right\rfloor}{\{x\}+2\left\lfloor x\right\rfloor}+\frac{\{x\}}{\left\lfloor x\right\rfloor+2\{x\}}\biggr)>\frac{14}{3} $

chiamo ora $ \left\lfloor x\right\rfloor=a\in\mathbb{N^{\neq}} $ e $ \{x\}=b\in (0,1) $ e divido per $ 2 $, ottenendo:

$ \displaystyle\frac{a}{b}+\frac{b}{a}+\frac{a}{2a+b}+\frac{b}{2b+a}>\frac{7}{3} $

ma come è noto: $ \frac{a}{b}+\frac{b}{a}=\frac{a^2+b^2}{ab}>\frac{2ab}{ab}=2 $ (l' uguaglianza non puo' valere per i vincoli del problema), resta quindi da dimostrare che:

$ \displaystyle\frac{a}{2a+b}+\frac{b}{2b+a}>\frac{1}{3} $

svolgendo i conti:
$ \displaystyle a^2+b^2+4ab>1/3(2a^2+2b^2+5ab) $
cioè
$ \displaystyle 1/3(a^2+b^2)+7/3ab>0 $
che è sicuramente vera dato che $ a $ e $ b $ sono positivi. . . può andare?
Spargi il defoliante
sulla cassa dirigente
[anonimo]
Gi8
Messaggi: 42
Iscritto il: 17 ago 2012, 12:04

Messaggio da Gi8 »

Se $ a \in (1,2) $ la disuguaglianza da dimostrare diventa $ \displaystyle \biggl(1+2\{a\} -\frac{1}{1+2\{a\}}\biggr)+\biggl(\frac{2+ \{a\}}{\{a\}}-\frac{\{ a\}}{2+\{a\}}\biggr) >\frac{14}{3} $
Faccio lo studio delle funzioni
$ \displaystyle f(x)= 1+2x - \frac{1}{1+2x} $ e $ \displaystyle g(x)= \frac{2}{x} +\frac{2}{2+x} $ con $ x \in (0,1) $
per concludere che $ \displaystyle \forall x \in (0,1) \qquad f(x)+g(x)>\frac{14}{3} $:
Testo nascosto:
  • $ \displaystyle f'(x) = 2+\frac{2}{(1+2x)^2}>0 \quad \forall x \in (0,1) $;
  • $ \displaystyle \lim_{x \to 0^+} f(x)= 0 $;
  • $ \displaystyle \lim_{x \to 1^-} f(x)= \frac{8}{3} $;
  • $ \displaystyle f\left(\frac{1}{2}\right)= \frac{3}{2} $;
  • $ \displaystyle f\left(\frac{3}{4}\right)= \frac{21}{10}>2 $;
Quindi
  • $ \displaystyle \forall x \in \left(0,\frac{1}{2} \right] \quad f(x)>0 $;
  • $ \displaystyle \forall x \in \left(\frac{1}{2},\frac{3}{4} \right] \quad f(x)> \frac{3}{2} $;
  • $ \displaystyle \forall x \in \left(\frac{3}{4} ,1\right) \quad f(x)>2 $;
Testo nascosto:
  • $ \displaystyle g'(x) = - \frac{2}{x^2} -\frac{2}{(x+2)^2}<0 \quad \forall x \in (0,1) $;
  • $ \displaystyle \lim_{x \to 0^+} g(x)= +\infty $;
  • $ \displaystyle \lim_{x \to 1^-} g(x)= \frac{8}{3} $;
  • $ \displaystyle g\left(\frac{1}{2}\right)= \frac{24}{5} $;
  • $ \displaystyle g\left(\frac{3}{4}\right)= \frac{8}{3}+\frac{8}{11} $;
Quindi
  • $ \displaystyle \forall x \in \left(0,\frac{1}{2} \right] \quad g(x)\geq \frac{24}{5} > \frac{14}{3} $ (poichè $\frac{4}{5}>\frac{2}{3}$);
  • $ \displaystyle \forall x \in \left(\frac{1}{2},\frac{3}{4} \right] \quad g(x) \geq \frac{8}{3}+\frac{8}{11} > \frac{8}{3}+\frac{1}{2} $;
  • $ \displaystyle \forall x \in \left(\frac{3}{4} ,1\right) \quad g(x)>\frac{8}{3} $;
Se $a$ è un numero reale non intero maggiore di $2$, si dimostra che $ \displaystyle\frac{a+\lfloor a\rfloor}{\{a\}}-\frac{\{ a\}}{a+\lfloor a\rfloor} >\frac{24}{5} $,
che mi permette di concludere in quanto $ \frac{a+\{a\}}{\lfloor a\rfloor}-\frac{\lfloor a\rfloor}{a+\{a\}} $ è sempre positivo.
Testo nascosto:
Consideriamo la funzione $ \displaystyle h(x) =x- \frac{1}{x} $ con $ x\in (1, +\infty) $.
Dato che $h'(x)= 1+\frac{1}{x^2}>0$ per ogni $x >1$ e $h\left( 5\right) = \frac{24}{5}$, mi basta dimostrare che $ \displaystyle \frac{a+ \lfloor a\rfloor}{\{a\}}>5 $.

Ma $ \displaystyle \frac{a+ \lfloor a\rfloor}{\{a\}}>5 \Leftrightarrow 1+ 2 \frac{\lfloor a \rfloor}{\{a\}}>5 \Leftrightarrow \frac{\lfloor a \rfloor}{\{a\}}>2 $.
Ora, poichè $0< \{a\}<1$ e $\lfloor a \rfloor\geq2$ si ha proprio $ \displaystyle \frac{\lfloor a \rfloor}{\{a\}}> \lfloor a \rfloor\geq 2 $. Fine
Simo_the_wolf
Moderatore
Messaggi: 1053
Iscritto il: 01 gen 1970, 01:00
Località: Pescara

Re: Disuguaglianza con [x] e {x}

Messaggio da Simo_the_wolf »

Uhm... dimostrare che la medesima quantità è $ > \frac {15} 3 $, e che il bound non si può migliorare...
Rispondi